Đến nội dung

xuanhoan23112002 nội dung

Có 95 mục bởi xuanhoan23112002 (Tìm giới hạn từ 29-04-2020)



Sắp theo                Sắp xếp  

#706708 [TOPIC] SỐ HỌC ÔN TẬP THPT CHUYÊN TOÁN 10 NĂM HỌC 2018-2019

Đã gửi bởi xuanhoan23112002 on 22-04-2018 - 23:08 trong Tài liệu - Đề thi

Bài 83: Giải phương trình nghiệm nguyên dương:

$x^3-(x+y+z)^2=(y+z)^3+34$




#706709 [TOPIC] ÔN THI BẤT ĐẲNG THỨC $\boxed{\text{THPT CHUYÊN}}$...

Đã gửi bởi xuanhoan23112002 on 22-04-2018 - 23:11 trong Tài liệu - Đề thi

Cách của bạn Linh đúng rồi mọi người thử tìm các cách khác chẳng hạn như dùng nguyên lí Đirichlet




#706710 [TOPIC] ÔN THI BẤT ĐẲNG THỨC $\boxed{\text{THPT CHUYÊN}}$...

Đã gửi bởi xuanhoan23112002 on 22-04-2018 - 23:14 trong Tài liệu - Đề thi

Bài 79: Cho a, b, c là các số thực dương thỏa mãn $a+b+c=3$. CMR:

$(a^2-a+1)(b^2-b+1)(c^2-c+1)\geq 1$




#706726 [TOPIC] ÔN THI BẤT ĐẲNG THỨC $\boxed{\text{THPT CHUYÊN}}$...

Đã gửi bởi xuanhoan23112002 on 23-04-2018 - 13:07 trong Tài liệu - Đề thi

Bài 81(VMO 2015): Cho a, b, c là các số thực dương. CMR:

$3(a^2+b^2+c^2)\geq (a+b+c)(\sqrt{ab}+\sqrt{bc}+\sqrt{ca})+(a-b)^2+(b-c)^2+(c-a)^2\geq (a+b+c)^2$




#706727 [TOPIC] SỐ HỌC ÔN TẬP THPT CHUYÊN TOÁN 10 NĂM HỌC 2018-2019

Đã gửi bởi xuanhoan23112002 on 23-04-2018 - 13:14 trong Tài liệu - Đề thi

Bài 85(VMO 2007): Cho x, y là các số nguyên, $x\neq -1, y\neq -1$ thoả mãn: $\frac{x^4-1}{y+1}+\frac{y^4-1}{x+1}$ là số nguyên. CMR

$x^{4}y^{44}-1$ chia hết cho x+1




#706767 [TOPIC] ÔN THI BẤT ĐẲNG THỨC $\boxed{\text{THPT CHUYÊN}}$...

Đã gửi bởi xuanhoan23112002 on 23-04-2018 - 19:52 trong Tài liệu - Đề thi

Bài 84: Cho $0< x, y, z< 1$ thỏa mãn: $xyz=(1-x)(1-y)(1-z)$. CMR: $x^2+y^2+z^2\geq \frac{3}{4}$




#706768 [TOPIC] ÔN THI BẤT ĐẲNG THỨC $\boxed{\text{THPT CHUYÊN}}$...

Đã gửi bởi xuanhoan23112002 on 23-04-2018 - 19:56 trong Tài liệu - Đề thi

Bài 85: Cho a, b, c là các số thực dương thỏa mãn $a^2+b^2+c^2+abc=4$. CMR: 

$a+b+c\geq \sqrt{a}+\sqrt{b}+\sqrt{c}$




#706771 [TOPIC] SỐ HỌC ÔN TẬP THPT CHUYÊN TOÁN 10 NĂM HỌC 2018-2019

Đã gửi bởi xuanhoan23112002 on 23-04-2018 - 20:09 trong Tài liệu - Đề thi

Bài 91: Tìm tất cả các cặp số (p, n) với p là số nguyên tố và n là số nguyên dương thỏa mãn:

$p^3-2p^2+p+1=3^n$




#706779 [TOPIC] SỐ HỌC ÔN TẬP THPT CHUYÊN TOÁN 10 NĂM HỌC 2018-2019

Đã gửi bởi xuanhoan23112002 on 23-04-2018 - 21:32 trong Tài liệu - Đề thi

Bài 94: Giải phương trình nghiệm nguyên: $x^3-x^2+8=y^2$




#707185 Tìm giá trị nhỏ nhất của: $S=a_{1}^{2}+a_{2...

Đã gửi bởi xuanhoan23112002 on 28-04-2018 - 15:19 trong Bất đẳng thức - Cực trị

Problem: Cho $a_{1}, a_{2},...,a_{19}$ là các số tự nhiên thỏa mãn: $a_{1}+a_{2}+...+a_{19}=26.$ Tìm giá trị nhỏ nhất của: $S=a_{1}^{2}+a_{2}^{2}+...+a_{19}^{2}.$

 




#707240 [TOPIC] ÔN THI BẤT ĐẲNG THỨC $\boxed{\text{THPT CHUYÊN}}$...

Đã gửi bởi xuanhoan23112002 on 29-04-2018 - 07:55 trong Tài liệu - Đề thi

Bài 106Cho a1, a2,...,a19 là các số tự nhiên thỏa mãn: a1+a2+...+a19 =26. Tìm giá trị nhỏ nhất của:

S=a12+a22+...+a192




#707774 ĐỀ THI OLYMPIC CHUYÊN KHTN 2018

Đã gửi bởi xuanhoan23112002 on 06-05-2018 - 16:28 trong Thi HSG cấp Tỉnh, Thành phố. Olympic 30-4. Đề thi và kiểm tra đội tuyển các cấp.

2 ngày thì mọi người làm đc mấy bài




#707928 Bất đẳng thức

Đã gửi bởi xuanhoan23112002 on 08-05-2018 - 21:35 trong Bất đẳng thức và cực trị

BĐT$\Leftrightarrow \frac{a^2}{ab+ac}+\frac{b^2}{bc+bd}+\frac{c^2}{cd+ca}+\frac{d^2}{da+db}\geq 2$

Ta có VT$\geq \frac{(a+b+c+d)^2}{(a+d)(b+c)+(c+d)(a+b)}$( theo BĐT Cauchy-Schwarz)

Mà cũng theo BĐT AM-GM ta cũng có $(a+d)(b+c)+(a+b)(c+d)\leq \frac{(a+b+c+d)^2}{2}$

Do đó VT$\geq 2$

Vậy bất đẳng thức được chứng minh. Đẳng thức xảy ra$\Leftrightarrow$ $a= b= c= d> 0$




#707935 [TOPIC] ÔN THI BẤT ĐẲNG THỨC $\boxed{\text{THPT CHUYÊN}}$...

Đã gửi bởi xuanhoan23112002 on 08-05-2018 - 22:54 trong Tài liệu - Đề thi

P/s: Topic dạo này buồn quá. Bài mới nha mọi người

Bài 123: Cho a, b, c là các số thực dương thỏa mãn: $\sqrt{a}+\sqrt{b}+\sqrt{c}\geq 1$. Chứng minh rằng:

$a^2+b^2+c^2+7(ab+bc+ca)\geq \sqrt{8(a+b)(b+c)(c+a)}$

 

Chúc 2k3 thi tốt bình tĩnh, tự tin, chiến thắng đạt được những mục tiêu đã đề ra!

:ukliam2:  :ukliam2:  :ukliam2:  :ukliam2:  :ukliam2:  :ukliam2:  :ukliam2:  :ukliam2:  :ukliam2:




#709289 Bất đẳng thức trong đề thi vào lớp 10 Nam Định năm 2018

Đã gửi bởi xuanhoan23112002 on 26-05-2018 - 15:38 trong Bất đẳng thức và cực trị

Cho a, b, c là các số thực dương thỏa mãn: $a^2+b^2+c^2+abc=4$. Chứng minh rằng:

$2a+b+c\leq \frac{9}{2}$




#709300 Bất đẳng thức trong đề thi vào lớp 10 Nam Định năm 2018

Đã gửi bởi xuanhoan23112002 on 26-05-2018 - 17:55 trong Bất đẳng thức và cực trị

Đây là đề của ban xã hội bạn ạ.

Còn đây là lời giải của mình cho bài toán này các bạn có thể tham khảo:

Coi phương trình trên là phương trình bậc 2 ẩn a theo công thức nghiệm ta được

$a=\frac{-bc+\sqrt{(4-b^2)(4-c^2)}}{2}$

Áp dụng bất đẳng thức Cauchy cho căn thức trong biểu thức trên, ta có:

$a\leq \frac{-bc+\frac{4-b^2+4-c^2}{2}}{2}= \frac{8-(b+c)^2}{4}$

Từ đó ta có: $2a+b+c=\frac{8-(b+c)^2+2(b+c)}{2}=\frac{9-(b+c-1)^2}{2}\leq \frac{9}{2}$

P/s: Mình nghĩ đây là cách ngắn nhất và có thể thay số 2 trong đề bài bởi các số khác vẫn có thể giải tương tự.




#709350 Đề thi vào 10 chuyên Lê Hồng Phong - Nam Định 2018 - 2019 - Toán Chuyên

Đã gửi bởi xuanhoan23112002 on 27-05-2018 - 10:49 trong Tài liệu - Đề thi

Câu 5:

a. Có $\frac{1}{2}\sqrt{(a+3b)(b+3a)}\leq \frac{a+3b+b+3a}{4}=a+b$ (bất đẳng thức AM-GM)

Từ giả thiết: $\sqrt{a}+\sqrt{b}=1$ 

Bình phương 2 vế ta có: $2\sqrt{ab}=1-a-b$ 

Hay $4ab=(1-a-b)^2$

Nên bất đẳng thức cần chứng minh tương đương với:

$3(a+b)^2+(1-a-b)^2\geq 2(a+b)$

$\Leftrightarrow (2a+2b-1)^2\geq 0$ (luôn đúng)

Vậy bất đẳng thức đã cho được chứng minh.




#709360 Cho $x, y, z > 0$ và $x+y+z= 1$ . Chứng minh

Đã gửi bởi xuanhoan23112002 on 27-05-2018 - 15:33 trong Bất đẳng thức và cực trị

Theo bất đẳng thức Schur ta có:

$(x+y+z)^3+9xyz\geq 4(x+y+z)(xy+yz+zx)$

$\Leftrightarrow 9xyz\geq 4(xy+yz+zx)-1$

$\Leftrightarrow 5xyz+1\geq 4(xy+yz+zx-xyz)$

Theo bất đẳng thức AM-GM ta có:

$xyz\leq \frac{(x+y+z)^3}{27}= \frac{1}{27}$

$\Rightarrow xy+yz+zx-xyz\leq \frac{8}{27}$

Vậy bất đẳng thức được chứng minh.




#709371 CMR: $\sum \frac{ab+c^{2}}{a+b}...

Đã gửi bởi xuanhoan23112002 on 27-05-2018 - 17:05 trong Bất đẳng thức và cực trị

Ta có: $\sum \frac{ab+c^2}{a+b}+\sum c= \sum \frac{(c+a)(c+b)}{a+b}\geq 2(a+b+c)$ (bất đẳng thức AM-GM)

$\Rightarrow \sum \frac{ab+c^2}{a+b}\geq a+b+c$

Đẳng thức xảy ra $\Leftrightarrow a=b=c> 0$

Vậy bất đẳng thức được chứng minh.




#709395 Đề thi vào trường chuyên Thái Bình năm 2019 (vòng 2)

Đã gửi bởi xuanhoan23112002 on 27-05-2018 - 20:34 trong Tài liệu - Đề thi

Câu 6:

Ta có:$\frac{1}{\sqrt{5a^2+2ab+2b^2}}=\frac{1}{\sqrt{(2a+b)^2+(a-b)^2}}\leq \frac{1}{2a+b}\leq \frac{1}{9}(\frac{2}{a}+\frac{1}{b})$ (bất đẳng thức Schwarz)

Chứng minh tương tự như trên ta có:

$P\leq \frac{1}{3a}+\frac{1}{3b}+\frac{1}{3c}$

Ta cũng có:$\frac{1}{a}+\frac{1}{b}+\frac{1}{c}\leq \sqrt{3(\frac{1}{a^2}+\frac{1}{b^2}+\frac{1}{c^2})}=\sqrt{3}$ (bất đẳng thức AM-GM)

Từ đó ta có: $P\leq \frac{\sqrt{3}}{3}$

Đẳng thức xảy ra $\Leftrightarrow a=b=c=\sqrt{3}$

Vậy MaxP = $\frac{\sqrt{3}}{3}\Leftrightarrow a=b=c=\sqrt{3}$.




#709478 [TOPIC] ÔN THI BẤT ĐẲNG THỨC $\boxed{\text{THPT CHUYÊN}}$...

Đã gửi bởi xuanhoan23112002 on 29-05-2018 - 08:41 trong Tài liệu - Đề thi

Bài 138: Với a, b, c là các số thực dương. Tìm giá trị nhỏ nhất của biểu thức:

$P=\frac{2}{a+\sqrt{ab}+\sqrt[3]{abc}}-\frac{3}{\sqrt{a+b+c}}$




#709479 Bài tập về đa thức

Đã gửi bởi xuanhoan23112002 on 29-05-2018 - 08:53 trong Đa thức

Bài 1: Cho đa thức $f(x)=x^{2018}+\sum a_ix^{i}($a_i\in {-1,1}, $\forall i\in \left \{ 0,1,...,2017 \right \}$$)$ không có nghiệm thực. Tìm số lớn nhất các hệ số = -1 trong f(x)

Bài 2: Tìm đa thức P(x) hệ số thực thỏa mãn:

$(P(x))^{3}-3(P(x))^{2}=P(x^{3})-3P(-x)$, với mọi x là số thực




#709502 Bất đẳng thức chọn lọc ôn chuyên

Đã gửi bởi xuanhoan23112002 on 29-05-2018 - 15:47 trong Bất đẳng thức và cực trị

Bài 3: Theo giả thiết ta có $0\leq a, b, c\leq 4$ nên

$$(4-a)(4-b)(4-c) \geq 0$$

$\Leftrightarrow 64+4(ab+bc+ca) \geq abc+16(a+b+c)$

$\Leftrightarrow ab+bc+ca\geq 8+\frac{abc}{4}\geq 8$ 

Do đó ta có: $P=(a+b+c)^2-(ab+bc+ca)\leq 36-8=28$

Đẳng thức xảy ra $\Leftrightarrow (a, b, c)=(0, 2, 4)$ và các hoán vị của nó

Vậy $MaxP=28$ $\Leftrightarrow (a, b, c)=(0, 2, 4)$ và các hoán vị của nó




#709553 số học

Đã gửi bởi xuanhoan23112002 on 30-05-2018 - 07:32 trong Số học

Từ giả thiết ta thấy ngay a, b, c đều là các số lẻ mà một số chính phương lẻ chia 8 dư 1

Từ nhận xét trên: $a^{30}+b^{4}+c^{2018}\equiv 3$ (mod 8)




#709566 Số chính phương

Đã gửi bởi xuanhoan23112002 on 30-05-2018 - 09:39 trong Số học

Bài toán này sử dụng phương pháp bước nhảy Viete. Các bài viết khác về bước nhảy Viete trên VMF

http://diendantoanho...ước-nhảy-viete/

Lời giải của bài toán trên bạn có thể tham khảo ở đây: http://math.stackexc...-its-an-integer